Strengthen Questions - - Question 21

G: The group of works exhibited in this year's Metropolitan Art Show reveals a bias in favor of photographers. Equal...

Kelsi-Grau August 15, 2018

Submitting vs Exhibiting

I marked answer choice B out because I thought that the fee referred to submitting their work, and there was an equal amount of each medium submitted, so it does not show a bias in what was actually exhibited in the show. Can someone clear this up for me?

Replies
Create a free account to read and take part in forum discussions.

Already have an account? log in

ddupray September 3, 2018

I have the same question!!

Mehran September 4, 2018

Hi @Kelsi-Grau, @ddupray, thanks for your posts.

All right: this stimulus presents a couple of different speakers, G and H. The question stem asks you to Strengthen G's argument, the conclusion of which is "the group of works exhibited in this year's Metropolitan Art Show reveals a bias in favor of photographers." What support does G provide for this claim of bias? (1) Equal numbers of photographers, sculptors, and painters submitted works that met the traditional criteria for the show, yet (2) more photographs were exhibited than either sculptures or paintings, even though (3) each artist was allowed to submit work in one medium only.

Answer choice (B) would strengthen the argument that there was bias in favor of photographers. Why? Because if the fee for entry for photographs is lower ($25) than for painting or sculpture ($75), then the selection of more photos than paintings or sculptures is NOT because the latter raised more money for the show. In other words, answer choice (B) strengthens the argument in the stimulus by ruling out an alternative cause (cash flow). This bolster's G's argument that the cause of the noticed effect (that more photographs were exhibited than either sculptures or paintings) was bias.

Hope this helps! Please let us know if you have any additional questions.

Samantha-Alexis June 2, 2019

Thank you for the explanation. I also had the same questions as the students below. I ended up narrowing the answer choices down to answers A and B and ultimately chose answer A, which was incorrect. Now, I understand how answer (B) rules out an alternate cause, but then how is answer (A) incorrect?

Ravi June 2, 2019

@Samantha-Alexis,

Great question.

(A) says, "If an artist has had one of his or her works exhibited in
the Metropolitan Art Show, that artist has an advantage in getting
commissions and selling works over artists who have never had a work
exhibited in the show."

The problem with (A) is that it gives us information about the
advantages of being in a show, but it does not give us any information
about how to get in the show. This is why (A) is incorrect.

Does this make sense? Let us know if you have any other questions!

grimadeau May 15, 2020

Can you explain alternate cause to me and how an answer choice is correct based on alter ante cause. I know that alternate cause weakens, so would be correct In weaken questions but how is alter ante cause relevant is strengthen ?

sangagul1234ah@gmail.com May 29, 2021

not an instructor but in a strengthen question if all other Alternative causes are ruled out then the argument is strengthened since there is no other cause but the one proposed by the argument.